Open zorkow opened 6 years ago
On this page: https://physics.stackexchange.com/questions/356216/derivation-of-the-klein-gordon-propagator Should be solvable with the new heuristics.
On this page: https://physics.stackexchange.com/questions/356216/derivation-of-the-klein-gordon-propagator Should be solvable with the new heuristics.